User avatar
 
ohthatpatrick
Thanks Received: 3808
Atticus Finch
Atticus Finch
 
Posts: 4661
Joined: April 01st, 2011
 
This post thanked 4 times.
 
 

Re: Q8 - This boulder is volcanic

by ohthatpatrick Fri Dec 31, 1999 8:00 pm

My concern with the way you worded your question is that you're under the impression that we're supposed to be able to predict this answer. I think Matt's post may have confused you by saying that (D) contradicts something that the argument is assuming.

That is true, but you shouldn't be thinking to yourself, "I should have been able to predict that ahead of time!" nor should you be thinking that THE assumption the argument makes is that there are some sources of volcanic boulders to the north. It is AN assumption the author makes, but potentially one of many.

There are plenty of LSAT arguments in which we see author's essentially doing "idea math" ... adding up one or two premises to try and reach a faulty conclusion, and in those arguments there's often just ONE missing piece/missing connection.

However, there are also PLENTY of arguments in which you're really dealing with something fuzzier like a prediction, or causality, or a suggested course of action ... these normally don't have answers that are very predictable because there are many possible ways to strengthen/weaken the argument (many different assumptions being made).

So you should never really think to yourself, "I missed the assumption." Most LSAT arguments have many, if not infinite, assumptions.

(The one exception to this is Sufficient Assumption and Principle-Justify. These two very similar question types are almost always testing one missing link.)

When you're doing Flaw/Nec Assump/Str/Weak, read the argument for missing logical links, but also open your mind up to the world of Potential Objections and Alternative Explanations.

The story this author is selling us on is this:
- we see a volcanic boulder, but nothing else around here is volcanic. How the heck did it get here? Why is it here?
- well, during the last ice age, a glacier north of here moved southward and covered this region. Maybe the volcanic boulder hitched a ride on the glacier and got dumped here. In other words, the volcanic boulder was born hundreds of miles to the north, and then the southward-moving glacier brought it down to this area and dumped it here.

I wouldn't have any specific prediction of what the correct answer is going to say/do, but what I would do is continually remind myself of the "anti-conclusion".

In order to weaken this argument, I want to be able to argue that this boulder "was probably NOT deposited here hundreds of miles from its geological birthplace, by a glacier."

Maybe the correct answer will help me argue that this boulder was NOT born hundreds of miles from here. Maybe the correct answer will help me argue that a southward-moving glacier was not the thing to deposit it here. Either one of those ideas would attack the validity of the conclusion.

(A) definitely weakens the argument somewhat. This is saying "your story doesn't apply in the case of most boulders. You think that a glacier moved this boulder hundreds of miles, but most of the time glaciers move a boulder it's 100 miles tops." Keep it.

(B) doesn't matter. The author isn't arguing that this volcanic boulder came from the closest source of volcanic rock. He's arguing that this boulder started up north and hitched a ride on a southward moving glacier. Eliminate.

(C) same issue as (B). We might say that this slightly weakens the claim that the boulder traveled hundreds of miles, because it's probably more common sense friendly to assume that if a glacier carried this volcanic boulder south that it did so from a closer distance. Maybe keep, but with a lot of hesitation.

(D) Game-changer. The author's story just crumbled. There is no way that this volcanic boulder was born in the north and hitched a ride south. There are NO sources of volcanic rock to the north. This is WAY stronger and thus more weakening than (A). Keep it and eliminate (A) and (C).

(E) If we thought that (C) mildly weakened by pointing out that "there are closer sources of volcanic boulders to the north than HUNDREDS of miles away" then this would mildly strengthen for the same reason.

Hope this helps.


#officialexplanation
 
chlqusghtk
Thanks Received: 0
Forum Guests
 
Posts: 17
Joined: September 18th, 2010
 
 
 

Q8 - This boulder is volcanic

by chlqusghtk Wed Sep 29, 2010 8:54 am

I got this right, and I totally understand why D is the answer. But, I'm still wondering why A cannot weaken the argument.

And, does C strengthen the argument? Or does it have no impact at all because it only deals with sources of rocks within 50 miles whereas the stimulus is considering the source which is hundreds of miles away?


Any help would be greatly appreciated.
User avatar
 
bbirdwell
Thanks Received: 864
Atticus Finch
Atticus Finch
 
Posts: 803
Joined: April 16th, 2009
 
 
 

Re: Q8 - This boulder is volcanic

by bbirdwell Fri Oct 01, 2010 5:14 pm

In regards to (A), it doesn't logically affect the argument, but it might help to think of it as weakening a teeny tiny bit because it simply says "most." That means 51%, which still means 49% of the boulders may have been moved more than 100 miles. Even if 99% of the boulders don't travel 100 miles, there's no information presented that would prevent this boulder from being the exception. You will occasionally see a choice like this. Helps to remember the question stem at that point and find the one that "most weakens."

I don't believe (C) has any impact at all. I think the author is trying to make you think that it weakens, though, in the sense of "why would the boulder come from hundreds of miles away if the closest source is 50 miles away?" But this imaginative thinking is incorrect: I could argue that my cowboy boots came from Argentina. You could say that there's a cowboy boot store down the street. So what?
I host free online workshop/Q&A sessions called Zen and the Art of LSAT. You can find upcoming dates here: http://www.manhattanlsat.com/zen-and-the-art.cfm
 
zainrizvi
Thanks Received: 16
Atticus Finch
Atticus Finch
 
Posts: 171
Joined: July 19th, 2011
 
 
trophy
First Responder
 

Re: Q8 - This boulder is volcanic

by zainrizvi Sun Nov 20, 2011 12:12 pm

q19-there-have-been-no-new-cases-t866.html

Can somebody help me reconcile this question with the one above?


In that one, the most allows us to get rid of choice (E) because it suggests that both vaccines suffer from the issue. So we assume that the most is all encompassing.

In this question, the most allows us to get rid of choice (A) because it suggests the possibility that this is somehow an exception. So we assume that the most is not all encompassing.


I can't really wrap my mind around this for some reason. Is it just because most is an "iffy" word, and depending on the context of other answer choices, its strength can vary?
 
slimjimsquinn
Thanks Received: 1
Forum Guests
 
Posts: 43
Joined: February 11th, 2012
 
 
 

Re: Q8 - This boulder is volcanic

by slimjimsquinn Wed Sep 12, 2012 6:10 pm

Why couldn't it be B)?

If the closest source of volcanic rock is 50 miles SOUTH of the boarder, how can a south moving glacier move the rock north to where the boulder was found?

I understand that D) is stronger but I wanted to see if there was any other reasoning to eliminate B).

Thanks!
 
jimmy902o
Thanks Received: 4
Elle Woods
Elle Woods
 
Posts: 90
Joined: August 06th, 2011
 
This post thanked 1 time.
 
 

Re: Q8 - This boulder is volcanic

by jimmy902o Wed Sep 12, 2012 8:43 pm

When i went through this test I also chose B too, but after looking back it says in the stim that the rock must have been deposited "hundreds of miles from its geological birthplace". So B says that the closest rock is 50 miles away but maybe another source of volcanic rock 55 or 60 miles north could have deposited. Given this info B would not weaken the stimulus. D on the other hand precludes any source north of the area and so is a stronger answer
 
alexg89
Thanks Received: 9
Jackie Chiles
Jackie Chiles
 
Posts: 39
Joined: July 24th, 2012
 
 
 

Re: Q8 - This boulder is volcanic

by alexg89 Sat Oct 20, 2012 4:48 pm

I do recall seeing a weaken question in an earlier preptest I think preptest 20 or lower range that had something like this as a correct answer so I agree that this is one of those cases where you need to choose the "best answer."

bbirdwell Wrote:In regards to (A), it doesn't logically affect the argument, but it might help to think of it as weakening a teeny tiny bit because it simply says "most." That means 51%, which still means 49% of the boulders may have been moved more than 100 miles. Even if 99% of the boulders don't travel 100 miles, there's no information presented that would prevent this boulder from being the exception. You will occasionally see a choice like this. Helps to remember the question stem at that point and find the one that "most weakens."

I don't believe (C) has any impact at all. I think the author is trying to make you think that it weakens, though, in the sense of "why would the boulder come from hundreds of miles away if the closest source is 50 miles away?" But this imaginative thinking is incorrect: I could argue that my cowboy boots came from Argentina. You could say that there's a cowboy boot store down the street. So what?
User avatar
 
ManhattanPrepLSAT1
Thanks Received: 1909
Atticus Finch
Atticus Finch
 
Posts: 2851
Joined: October 07th, 2009
 
This post thanked 3 times.
 
 

Re: Q8 - This boulder is volcanic

by ManhattanPrepLSAT1 Tue Nov 27, 2012 2:45 pm

We're asked to weaken the argument. It attempts to show that the rock was deposited in its position by a southward-moving glacier. Why? The boulder is volcanic in origin, surrounded by sedimentary rocks, and we know the area was covered by southward-moving glaciers during the last ice-age.

What's the argument assuming? It assumes that there are volcanic sources northward of the deposited rock. Answer choice (D) directly attacks this assumption and so weakens the argument.

Incorrect Answers
(A) leaves room for some boulders to have been moved by more than 100 miles and so does not challenge the argument's conclusion.
(B) leaves room for there to be volcanic sources to the north of this deposited boulder from which this boulder came.
Image
(C) leaves room for there to be volcanic sources hundreds of miles north of the deposited boulder from which this boulder came.
(E) leaves room for this boulder to have been deposited by a southward-moving glacier from a geologic source hundreds of miles north of the deposited boulder.
 
samiraa180
Thanks Received: 0
Vinny Gambini
Vinny Gambini
 
Posts: 14
Joined: April 07th, 2014
 
 
 

Re: Q8 - This boulder is volcanic

by samiraa180 Thu May 01, 2014 9:46 pm

So, I read the explanation provided above, and I still have a hard time grasping the argument, can you give me guidance on how to predict the assumption?

Conclusion: this boulder was probably deposited by a glacier. Why?

Evidence: it's volcanic in origin

Evidence: area is covered by southward-moving glaciers during the last ice age.

So when the evidence says that the area is covered by southward- moving glacier, is it talking about the original place of the boulder or where it moved to?

Assuming that my identification of the core is correct, how do I then identify the missing piece?
 
samiraa180
Thanks Received: 0
Vinny Gambini
Vinny Gambini
 
Posts: 14
Joined: April 07th, 2014
 
 
 

Re: Q8 - This boulder is volcanic

by samiraa180 Wed May 14, 2014 1:06 am

Yep! Thank you for going into that lengthy explanation.
 
jm.kahn
Thanks Received: 10
Elle Woods
Elle Woods
 
Posts: 88
Joined: September 02nd, 2013
 
 
 

Re: Q8 - This boulder is volcanic

by jm.kahn Sun May 03, 2015 7:17 pm

Earlier posts have tried to explain away A by saying that "most" in A doesn't include all boulders. But that expln doesn't seem right.

conclusion: the boulder was probably deposited here, hundred of miles from its birthplace, by a glacier

A says that "most boulders moved by glaciers aren't moved >100 miles." This boulder well may be the exception but A still undermines the possibility that the boulder was moved by glacier.

It also wouldn't be right to say that A weakens by a lesser extent than D. Only one answer choice weakens on the LSAT for such questions and that is the credited choice. All others must not weaken. So why doesn't A weaken at all?
 
jm.kahn
Thanks Received: 10
Elle Woods
Elle Woods
 
Posts: 88
Joined: September 02nd, 2013
 
 
 

Re: Q8 - This boulder is volcanic

by jm.kahn Sun May 03, 2015 7:58 pm

zainrizvi Wrote:http://www.manhattanlsat.com/forums/q19-there-have-been-no-new-cases-t866.html

Can somebody help me reconcile this question with the one above?

In that one, the most allows us to get rid of choice (E) because it suggests that both vaccines suffer from the issue. So we assume that the most is all encompassing.

In this question, the most allows us to get rid of choice (A) because it suggests the possibility that this is somehow an exception. So we assume that the most is not all encompassing.

I can't really wrap my mind around this for some reason. Is it just because most is an "iffy" word, and depending on the context of other answer choices, its strength can vary?


Interested in this as well



Also how does the analysis above by ohthatpatrick square with using "regularly" as a weakner in Q3 of the same section?

here is the quote from 56. LR1. Q3 page:

Note, that even believing she MIGHT be one of these top graduates is enough to cast doubt on the original argument (and "cast doubt" is equivalent to "weaken").


https://www.manhattanprep.com/lsat/foru ... t5179.html

In the 56.LR1.Q3 we use "regularly" to assume that it applies to Ms Garon's case. But in this case, we don't use "most" in choice-A to apply to the deposited boulder.
 
mshinners
Thanks Received: 135
Atticus Finch
Atticus Finch
 
Posts: 367
Joined: March 17th, 2014
Location: New York City
 
This post thanked 1 time.
 
 

Re: Q8 - This boulder is volcanic

by mshinners Tue Aug 25, 2015 10:55 am

jm.kahn Wrote:So why doesn't A weaken at all?


We have a volcanic boulder, while answer choice (A) talks about all boulders. It's entirely possible that most boulders don't move that much, but there's something unique about volcanic boulders that means they always get moved more by glaciers. So while most boulders don't move more than 100 miles, 100% of volcanic boulders have been moved that much.

This explanation is meant to highlight why a "most" statement almost always has no impact an argument that's talking about a specific example - the specific example we have could, in some way, differ from most other examples, thus resulting in a statement that doesn't affect the argument. Even if we stabilized (A) by talking about volcanic boulders, we'd have issues saying that it necessarily applied to this case.
 
mshinners
Thanks Received: 135
Atticus Finch
Atticus Finch
 
Posts: 367
Joined: March 17th, 2014
Location: New York City
 
 
 

Re: Q8 - This boulder is volcanic

by mshinners Tue Aug 25, 2015 11:01 am

zainrizvi Wrote:In that one, the most allows us to get rid of choice (E) because it suggests that both vaccines suffer from the issue. So we assume that the most is all encompassing.


I don't think the "most" is what lets us get rid of (E) (for the same reason that we can get rid of (A) here) - I think the rest of the answer does. The conclusion is that we should switch from OPV to IPV as the most commonly used polio vaccine - not as the only polio vaccine we use. Let's keep using OPV for children with neurological diseases, but switch to IPV for other cases.

That said, I think there's a bigger "takeaway" for this question - if you're looking to find an answer for a question that is based on a comparison, you should prefer an answer that brings up both of the things you're comparing. If I'm trying to say that IPV is safer than OPV, then I'd really like to know something that is safe about IPV AND doesn't apply to OPV. Otherwise, it might be a pro of both.
 
keonheecho
Thanks Received: 0
Elle Woods
Elle Woods
 
Posts: 54
Joined: August 20th, 2014
 
 
 

Re: Q8 - This boulder is volcanic

by keonheecho Sat Sep 05, 2015 10:41 am

Hi, I have a question about answer choice D.

I see how it could be argued that this answer weakens the assumption, but the answer choice says that there ARE no geological sources. But isn't it also possible that there once were geological sources that are no longer intact?

I get that A is incorrect because we would have to make certain unwarranted assumptions, but with D it also seems like we would have to assume that what happened now has been the case in the past...Is this true? And if it is, how can we discern which one is correct out of the two flawed answers? Thank you!
 
mshinners
Thanks Received: 135
Atticus Finch
Atticus Finch
 
Posts: 367
Joined: March 17th, 2014
Location: New York City
 
 
 

Re: Q8 - This boulder is volcanic

by mshinners Wed Apr 20, 2016 10:27 am

keonheecho Wrote:Hi, I have a question about answer choice D.

I see how it could be argued that this answer weakens the assumption, but the answer choice says that there ARE no geological sources. But isn't it also possible that there once were geological sources that are no longer intact?

I get that A is incorrect because we would have to make certain unwarranted assumptions, but with D it also seems like we would have to assume that what happened now has been the case in the past...Is this true? And if it is, how can we discern which one is correct out of the two flawed answers? Thank you!


Definitely true, and why (D) doesn't kill the argument. However, it does weaken it. Once you know that there are not currently sources, it sets up the presumption based on this fact, and flipping it to the other side (there weren't sources at some point) becomes the wild assumption instead of the better inference.